You are on page 1of 29

TRƯỜNG ĐẠI HỌC SƯ PHẠM HÀ NỘI

*****************

CHAPTER 3:
Numerical Sequences and Series

Người thực hiện _ Nhóm 2:


- Hoàng Minh Đạt
- Lý Thị Kiều Diễm
- Phạm Thùy Dương
- Trịnh Đăng Dương
- Nguyễn Hải Hà Giang
- Đoàn Đức Hà

Lớp CLC – K69 – Khoa Toán - Tin

HÀ NỘI – THÁNG 10/2020


0
Câu 3.1. CMR: Dãy S n  hội tụ thì dãy  S n  hội tụ. Điều ngược lại có đúng không?
Lời giải

 S n  hội tụ, ta sẽ chứng minh  S n  hội tụ.


Đặt lim Sn  c , với   0 tùy ý, M  thỏa mãn: S n  c   , n  M
n 


 Sn  c  Sn  c
Ta có:   Sn  c  Sn  c  Sn  c   , n  M
 S
 n  c  c  S n

  S n  hội tụ tới c
 Điều ngược lại không đúng.
Ví dụ: Dãy   1  hội tụ nhưng dãy  1  phân kì.
n n

Câu 3.2. Tìm lim  n  n  n  .


2
n 

Lời giải

  n
2
n2  n 2

 lim  n  n  n   lim
n 1 1 1
2
 lim  lim  
n  n 
n n n
2 n 
n n n
2 n  1 11 2
1 1
n
Câu 3.3. S1  2 và Sn 1  2  Sn  n  1, 2,... . CMR dãy S n  hội tụ và S n  2, n  1, 2...
Lời giải

 Ta chứng minh: Sn  2, n  1, 2,... .

Quy nạp:

Với n  1 , S1  2  2

Giả sử đúng với n  k : Sk  2 k   *



Với n  k  1 . Ta cần chứng minh Sk 1  2

Thật vậy, Sk 1  2  sk  2  2  2  2  2 .

Vậy Sn  2, n  1, n .

 Ta chứng minh S n  hội tụ  Dãy tăng bị chặn. Vì thế, ta chứng minh dãy tăng:
S n 1  S n  x  *
.
Quy nạp: 2  S1  S2  2  2

1
Giả sử đúng với n  k k   *
, S k 1  Sk .

Với n  k  1 , ta cần chứng minh Sk  Sk 1

Sk  2  Sk 1  2  S k  S k 1

Vậy S n 1  S n , n  *
. Dãy S n  là dãy tăng, và bị chặn trên bởi 2  S n  hội tụ.

Câu 3.4. Tìm giới hạn trên và giới hạn dưới của dãy S n  xác định bởi
S2 m 1 1
S1  0; S2 m  ; S2 m 1   S2 m .
2 2
Lời giải

1 1 1
 Từ giả thiết  S2 m1   S 2m1  S 2m 1  1   S 2m 1  1
2 2 2
1
2
 1
2
  2
1
2
1
 S2 m1  1  S2 m11  1  2 S2 m21  1  ...  m  S1  1   m

1 1
 S2 m1  1  m  lim S2 m1  1 (vì lim m  0 )
2 m  m 2
1 1 1 1  1 1 1
 S2 m  S2 m1  S2 m11  1  m1    m  lim S2 m 
2 2 2 2  2 2 m  2
1 1
Vậy S n  có các giới hạn riêng là 1 và  S *  1; S*  .
2 2
Câu 3.5. Cho hai dãy số thực an  và bn  . CMR: lim sup  an  bn   lim sup an  lim supbn .
n  n  n 

Lời giải

Do an  và bn  có vài trò bình đẳng nên ta chỉ xét 3 trường hợp sau:

 Trường hợp 1: limsup an   hoặc limsup bn  

 limsup an  limsup bn   (Do không xảy ra trường hợp    )

 lim sup  an  bn   lim sup an  lim sup bn

 Trường hợp 2: limsup an   và limsup bn  

 bn bị chặn trên bởi M .

Với x  , N 0 thỏa mãn n  N0 : an  x  an  bn  an  M  x  M

 lim sup  an  bn     lim sup an  lim sup bn

 Trường hợp 3: lim sup an và limsup bn là hữu hạn: Đặt lim sup an  a , limsup bn  b

2
+) Tồn tại dãy con ank  bnk   mà lim  a
k 
nk 
 bnk  lim sup  an  bn 
n 

Nếu limsup  an  bn   a  b  x0 thỏa mãn lim sup ank  bnk  x0  a  b


k 
 
+) N thỏa mãn k  N thì ank  bnk  x0 (*)

x0 a  b
Do   a  lim sup an
2 2

x0 a  b
 N1 thỏa mãn k  N1 thì ank  
2 2

x0 b  a
Do   b  lim sup bn
2 2

x0 b  a
 N2 thỏa mãn k  N 2 thì bnk  
2 2

 x0 a  b
ank  2  2
Chọn N 3  max  N1, N 2 , N   Với k  N 3 thì   ank  bnk  x0 , k  N 3  N
b  x0  b  a
 nk 2 2

(**)

 
Từ (*) và (**)  Mâu thuẫn  limsup ank  bnk  a  b  limsup an limsup bn .

Câu 3.6. Kiểm tra sự hội tụ của những chuỗi sau:


Lời giải

a) an  n  1  n

m
  an  m  1  1
n 1

  an  lim
m 
 
m  1  1  

  an phân kì.

n 1  n 1 1 1
b) an     3
n n  n 1  n  2n n
2n 2

1
Do  3
hội tụ và an  0, n  .
2
2n

3
  an hội tụ.

 
n
c) an  n
n 1

 lim n an  lim n n  1  1  1  0
n  n 

  an hội tụ.

1 1
d) Nếu z  1  an    lim an  0   an phân kì.
1 z n
1 zn n 

1 1 1 z 1
Nếu z  1   n  n n 1
 . n
1 z n
z 1 z  z z 1 z

z 1
. n hội tụ  
1
Do hội tụ.
z 1 z 1 zn

an
Câu 3.7. a n hội tụ , an  0 . Chứng minh rằng  n
hội tụ.

Lời giải
2
 1
 Ta có:  an    0
 n

2 an 1 an an 1
 an   2 0   2
n n n 2 2n

an 1 1 an
Mà 2 
2
 an hội tụ và  2n 2
hội tụ nên  n
hội tụ.

Câu 3.8. Nếu a n hội tụ và nếu dãy bn  là đơn điệu, bị chặn. CMR a bn n hội tụ.
Lời giải

a n hội tụ, bn  đơn điệu và bị chặn. Chứng minh  a .b


n n hội tụ.

n
Gọi An   ak là dãy tổng riêng thứ n .
k 1

  An  bị chặn  An  M , n

Giả sử bn  dãy đơn điệu tăng, bị chặn trên   lim bn  b  sup bn  .
n 

Đặt  n  b  bn  0   n   m , n  m

4

   0, N  0, n  N : 0   n 
2M

Xét n  N , p  

an 1 n 1  an  2 n  2  ...  an  p n  p   An 1  An   n 1   An  2  An 1   n  2  ...   An  p  An  p 1   n  p

  n 1 An   n 1   n  2  An 1   n  2   n 3  An  2  ...   n  p 1   n  p  An  p 1   n  p An  p

 M  n 1   n 1   n  2    n  2   n 3   ...   n  p 1   n  p    n  p 


 M .2 n 1  2M 
2M

 an 1 n 1  an  2 n  2  ...  an  p n  p     an n hội tụ.

Mà a  n n   an  b  bn   b an   anbn

Lại có: b an hội tụ và a  n n hội tụ nên a b n n hội tụ.

Câu 3.9. Tìm bán kính hội tụ cho từng chuỗi hàm sau:
Lời giải

a) n z 3 n

3
a n 1
Cách 1: Ta có: an  n . Xét lim n  lim
3
 lim 1
 n  1 n 1  1 
n  a n  3 3
n 1
 
 n

  n3 z n có bán kính hội tụ R  1

1 1
Cách 2: R   1
lim n an lim n n3
n  n 

2n n
b)  n ! .z
2n a 2n  n  1 ! n 1
Ta có: an  . Xét lim n  lim . n 1  lim 
n! n  a n  n ! 2 n  2
n 1

2n n
 .z có bán kính hội tụ R  
n!

5
2n n
c)  n2 z

2n  n  1  n  1  lim  n  1  1
2 2 2
2n a
Ta có: an  2 . Xét lim n  lim 2 . n 1  lim
n n  a
n 1
n  n 2 n  2n 2 n  2n 2 2

2n n 1
 2
z có bán kính hội tụ R 
n 2

n3
d) 3 n
.z n

n3 an n3 3n1
Ta có: an  . Xét lim  lim . 3
 n  1
n  3n 3
3n n  a
n 1

n3 n
  n .z có bán kính hội tụ R  3
3

Câu 3.10. Giả sử rằng hệ số của chuỗi hàm sau a z n


n
đều nguyên, trong đó có vô số hệ số khác
0. Chứng minh bán kính hội tụ của chuỗi hàm đó cùng lắm là 1.
Lời giải

Ta có: an  , n 

an  0 tại vô hạn n

  dãy con ank   mà a nk  


, k  

1
 nk ank  1    lim sup n an  1  R  1.
n  

Vậy bán kính hội tụ R  1.

Câu 3.11. Giả sử an  0 , sn  a1  a2  a3  ...  an và a n phân kì.


an
a) Chứng minh rằng  1 a phân kì.
n

aN 1 a S aN
b) Chứng minh rằng
S N 1
 ...  N  k  1  N . Và suy ra rằng
SN k SN k
S phân kì.
N

an 1 1 an
c) Chứng minh rằng
Sn 2
  . Và suy ra rằng
S n 1 S n
S 2
hội tụ.
n

an an
d) Có thể nói gì về  1  na và 1 n a 2
?
n n

6
Lời giải

an
a) CMR:  1 a phân kì.
n

an a
Nếu an  không bị chặn, tức lim an   . Khi đó, lim  1  0   n phân kì.
n  n  1  an 1  an

an a
Nếu an  bị chặn  M  0 sao cho an  M khi n   và  n .
1  an 1  M

an
Mà a phân kì   phân kì.
1  an
n

aN 1 a a  ...  aN  k S N  k  S N S
b) Ta có:  ...  N  k  N 1   1 N
S N 1 SN k SN k S N k S N k

Vì cho dù N rất lớn nhưng nếu N cố định, ta hoàn toàn có thể lấy k đủ lớn để
SN 1 S 1
 (Vì SN k   khi k   ), tức 1  N  .
S N k 2 SN k 2

aN
Do đó, tổng riêng của S không lập thành dãy Cauchy.
N

aN
 phân kì.
SN

1 1 Sn  Sn 1 an a
c) Ta có:     n2
Sn 1 Sn Sn .Sn 1 Sn .Sn 1 S n


 1 1  1 a
Vì  S   hội tụ đến   n2 hội tụ.
n2  n 1 Sn  a1 Sn

an
d) Xét  1 a
n

Nhận xét: Tổng này có thể hội tụ hoặc phân kì.

 Giả sử: nan  hoặc bị chặn trên hoặc bị chặn dưới bởi một số nguyên dương thì
an
 1 a phân kì. Thật vậy,
n

an 1 an
+) Nếu nan  M  
1  nan 1  M
an mà a n phân kì nên  1  na phân kì.
n

an  1
+) Nếu nan    0   .
1  nan 1   n
7
an  1 1 an
Do đó,  1  na 
1 
n mà n phân kì 
1  nan
phân kì.
n

an
 Tuy nhiên  1  na cũng có thể hội tụ.
n

1  n CP 

Ta chọn dãy an   1 (CP: chính phương k CP: không chính phương)
 2  n k CP 
n

ới M  *
t y
1  M  M
M M   M  
an 1 1 1
   n2    
n 1 1  n.an n 1 1  n n 1 n  n n 1 1  n
2 2 2
1
n 1
1  n. 2
n
 
1 1
Do n
n 1
2
; 
 n n 1 1  n 2
hội tụ nên bị chặn

M
an
 bị chặn
n 1 1  nan


an
 hội tụ.
n 1 1  nan

an
 So với 1 n a 2
n

an a 1 1 an
Ta có:  2 n  2 mà
1  n an n an n
2 n 2
hội tụ nên 1 n a 2
hội tụ.
n


Câu 3.12. Giả sử an  0 và  an hội tụ. Đặt rn   an mn

am a r an
a) Chứng minh rằng
rm
 ...  n  1  n . Nếu m  n và suy ra rằng
rn rm
s phân kì.
n

b) Chứng minh rằng


an
rn
2  
rn  rn 1 . Và suy ra rằng 
an
rn
hội tụ.

Lời giải

a) an  0 , a n hội tụ, rn   am  m  n  rm  rn 
mn

Nhận xét: Với p, q  , p  q thì rp  rq .

Khi đó, với m  n


8
am a a  ...  an rm  rn 1 r r
 ...  n  m   1  n 1  1  n (Do rn 1  rn )
rm rn rm rm rm rm
n
ak r
  1 n
k  m rk rm

 r  a
Nhận xét: lim 1  n   1 (Do rn  0 khi n   )   n không thỏa mãn tính chất Cauchy
n 
 rm  sn

an
 phân kì.
sn

b) Ta có:

an
rn
2  rn  rn 1 

an
rn
 
rn  rn 1  2  rn  rn 1 

 r 
 an 1  n 1   2an
 rn 
 
rn 1
 1 2
rn
 rn 1  rn

(luôn đúng)

Khi đó

   
N N
an
SN    2 rn  rn 1  2 r1  r2  r2  r3  ...  rN  rN 1  2 r1  2 rN 1
n 1 rn n 1

 lim Sn  2 r1  2 a1
n 

an
 hội tụ (đpcm).
rn

Câu 3.13.  a ;b


n n hội tụ tuyệt đối. CMR:   a  .   b  cũng hội tụ tuyệt đối.
n n

Lời giải

Từ giả thiết ta có  a ;b n n là các chuỗi số dương.

Khi đó ta cần chứng minh   a  .   b  hội tụ.


n n

9
n n n k
Đặt Sn   an ; Tn   bn ;U n   al bk l vì Sn ; Tn hội tụ; Sn ; Tn tăng. ậy Sn ; Tn bị chặn, coi
k 0 k 0 k  0 l 0

a1  T1  0 .
n k n k
U n   al bk l   al Tk l  Tk l 1 
k 0 l 0 k 0 l 0

  ak  j T j  T j 1 
n k

k 0 j 0

   ak  j  ak  j 1  .T j
n k

k 0 j 0

   ak  j  ak  j 1  .T j
n n

j 0 k  j
n n
  an  j .T j  T . am  T .S n  T .S
j 0 m0

Như vậy, U n tăng và bị chặn. Do đó U n hội tụ.

Câu 3.14. Nếu sn  là một dãy phức, định nghĩa số học của nó  n là:
s0  s1  ...  sn
n 
n1
 n  0,1, 2,...
a) Nếu lim sn  s . Chứng minh rằng lim  n  s .
b) Xây dựng một dãy sn  không hội tụ nhưng lim  n  0 .
c) Có thể xảy ra trường hợp sn  0, n và lim sup sn   nhưng lim  n  0 không?
1 n
d) Đặt an  sn  sn 1 với n  1 . Chứng minh rằng sn   n   ka . Giả sử lim  nan   0 và
n  1 k 1 k
  hội tụ. Chứng minh s  hội tụ.
n n

e) Rút ra kết luận cuối cùng từ một giả thuyết yếu hơn: Giả sử M   , nan  M với mọi n , và
lim  n   . Chứng minh rằng lim sn   .

Lời giải


a) Vì lim sn  s nên   0 , N1 : sn  s  , n  N 1 .
2
Vì sn  là dãy hội tụ trong không gian metric nên nó bị chặn (vì mọi dãy hội tụ trong không
gian metric là bị chặn).
 M  0 : sn  s  M , n  N1 .

s0  s1  ...sn s  s  ...  sn  s
Ta có  n  s  s  0
n1 n1

10
s0  s  ...  sN1 1  s sN1  s  ...  sn  s
 
n1 n1

N1 . M n  N 1
 1 .
2  N1 M  
 
n1 n1 n1 2
2 N1 M
Chọn N 2 là số nguyên dương nhỏ nhất thỏa mãn N 2  1

  N2  1 N1 M   N2  1 NM 
 2 N1 M     N 2  N 1 M     1  , n  N 2 .
2 n1 2  n  1 n1 2
N1 M   
Đặt N  max  N1 , N 2    n  s       , n  N ,
n1 2 2 2
 lim  n  s .
 1
s0  s1  ...  sn  , n  2 k( k  )
b) Xét sn   1 , khi đó  n 
n
 n  1  lim  n  0 .
n1 0, n  2 k  1( k  )

Tuy nhiên sn  không hội tụ.

1 n  0

1

c) Xét sn    3 n n  k 3 , k  
 thì s
n
 0, n  .
n
1
 n 
n  k3 , k  
Số lượng số lập phương trong 1, 2,...,n là  3 n  .
 
1 3  3 2
s  s  ...  sn 1  n.  n . n
n   2  n3
n  0 1   0
n1 n1 n1
1 
lim sup sn  lim   3 n   
n n
 
d) Chứng minh bằng quy nạp theo n .
s s s s 1
+) n  1 : s1   1  s1  0 1  1 0  a1 .
2 2 2
Giả sử kết luận đúng đến n  n  1 .

1 n 1 1  n 
Ta có:  ka    kak   n  1 sn1  sn  
n  2 k 1 k
n  2  k 1 


1
n2

 n  1 sn   n    n  1 sn1  sn  

1
n2
 n  1 sn1   n 

11

 n  1 sn 1
  s0  ...  sn 
n2


 n  2  sn1   s0  ...  sn1 
n2
s  ...  sn1
 sn1  0  sn1   n1  đpcm.
n2
Từ câu a), thay sn  bởi nan   lim  sn   n   0 .

 sn  hội tụ.


s0  s1  ...  sn s0  ...  sm
e) Nếu m  n ta có:  n   m  
n1 m1
mn n
si
  s0  ...  sn  .  
 m  1 n  1 im1 m  1
mn 1 n

m1
. n  s
m  1 i  m 1 i
m1 n
m1 n

mn
 n  m   n 
1
 s
m  n i  m 1 i
  n
      
1
s
m  n n m m  n i  m 1 i
m1 n
 sn   n       
1
 s  s 
n  m n m n  m i  m 1 n i
 1 1   n  i  M  n  m  1 M
Có sn  si  ai 1  ...  an  M   ...    
 i 1 n i 1 m2
n x n1
(do hàm   1 giảm)
x1 x1
Cố định   0 , với mỗi n chọn m nguyên dương sao cho
n
m  m1
1
n  m  1
 m  2  

m1  1
 n  m 
n
Chọn m là số nguyên lớn nhất trong các số , rõ ràng m  n .
1
Vì  cố định nên ta có thể giả sử m   .
n n m1
Có:  m1 
1 m2
n m1 1
m  
1  nm 
1
m, n   : sn   n   n   m  M  lim sn   n  0

 lim sn  lim  n   .
12
Câu 3.15. Chứng minh rằng các định lí 3.22 , 3.23 , 3.25  a  , 3.33 , 3.34 , 3.42 , 3.45 , 3.47 và 3.55
k
đúng trong trường hợp mở rộng an nằm trong .
Lời giải
m
Định lí 3.22 mở rộng: an  k
, a n
hội tụ    0, N thỏa mãn a
k n
k
  , m  n  N .

Chứng minh:
n
   : Đặt s   a
n k
suy ra a n
hội tụ.
k 1

 sn  là dãy Cauchy.

   0, N thỏa mãn m  N , n  N , sm  sn  

m m n 1
 m  n  N  1 : a
k n
k
 a a
k 1
k
k 1
k
 sm  sn1   .

m
   :   0, N thỏa mãn  a k
  , m  n  N .
k n

Nếu m  n  sm  sn  0   .

m
Nếu m  n  a
k  n 1
k
 sm  sn  

 sm  sn   , m  n  N  sn  là dãy Cauchy   an hội tụ.

Định lí 3.23 mở rộng: an 


k
, nếu a n hội tụ  lim an  0 .
n

Chứng minh:


Đặt an  an , an ,..., an .
1 2 k

a n hội tụ  với mỗi j  1, 2,...,k , an 1


nj hội tụ  lim an  0  lim an  0 .
n j n

Định lí 3.25 mở rộng: an  , cn 


k
.

Nếu an  cn , n  N 0 , c n hội tụ thì a n hội tụ.

Chứng minh:

Với mỗi j  1, 2,...,k : an  cn , n  N 0 .


j

13
Do c n
hội tụ suy ra a nj
hội tụ suy ra a
n
hội tụ.

Định lí 3.33 mở rộng: an  k


,   lim sup n an .
n

a) Nếu   1 thì a n
hội tụ.
b) Nếu   1 thì  a n
phân kì.
c) Nếu   1 thì không xác định được.

Chứng minh:

a) Với n tùy ý, n anj  n an ,  j  1, 2,..., k .

 lim sup n anj  1

  anj hội tụ   an hội tụ.

b)   1  an  1 tại vô hạn n .

 lim an  0   an không hội tụ.


n

1 1 1
c) Ví dụ: an   , ,...,    an phân kì.
n n n

 1 1 1 
an   2 , 2 ,..., 2    an hội tụ.
n n n 

Định lí 3.34 mở rộng: an 


k
:

an1
a) a n hội tụ nếu lim sup
n an
 1.

an1
b) a n phân kì nếu
an
 1, n  n0 .

Chứng minh:

an1
a) lim sup 1
n an

an1
 r thỏa mãn lim sup r 1
an

an1
 N thỏa mãn  r , n  M
an
14
 an1  an .r , n  M

 an  aM .r n M , n  M


Do a
n M
M
.r n M hội tụ   an hội tụ.

an1
b)  1, n  n0  an  an0 , n  n0
an

 lim an  an0  0  lim an  0   an phân kì.


n

Định lí 3.42 mở rộng: an  k


. Giả sử ta có:

a) Tổng riêng An bị chặn.


b) b0  b1  b2  ...
c) lim bn  0 .
n

Thì a b n n hội tụ.

Chứng minh:


An  An1 , An2 ,..., Ank 
Với i  1,2,..., k :
n
Ani   a ji  Ani bị chặn.
j 1


  ani .bni hội tụ.
n 1

  an bn hội tụ.

Định lí 3.47 mở rộng: an


 A ,  bn  B a ,b
n n
 k

   an  bn   A  B và  c.a n
 c. A .

Chứng minh:

A   A1 , A2 ,..., Ak  , B   B1 , B2 ,..., Bk  .

Với i  1,2,..., k :

a ni 
 bni   ani   bni  Ai  Bi

15
   an  bn   A  B .

 c.a ni
 c. ani  c.Ai

  c.an  c.A .

Định lí 3.55 mở rộng: a n


hội tụ tuyệt đối suy ra tồn tại sắp xếp lại của a n
hội tụ và hội tụ
tới cùng một giá trị.

Chứng minh:

a n
hội tụ tuyệt đối suy ra a ni hội tụ tuyệt đối

 mọi sự sắp xếp lại của a ni đều hội tụ tới một giá trị.

 mọi sự sắp xếp lại của a n


đều hội tụ tới một giá trị.

Câu 3.16. Lấy một số dương  . Chọn x1   , và định nghĩa x2 , x3 , x4 ,... , bởi công thức đệ quy:
1  
xn1   xn   .
2 xn 
a) Chứng minh rằng dãy xn  giảm đơn điệu và lim xn  a .

 n2  n2
b) Lấy  n  xn   , có  n 1   , vậy nên, đặt  2  ,
2 xn 2 
2n
 
 n 1    1   n  1,2,3,...  .

c) Đây là một thuật toán tốt trong việc tính giá trị của căn thức, vì công thức đệ quy đơn giản
1 1
và hội tụ cực kì nhanh chóng. Ví dụ, nếu   3 và x1  2 , chỉ ra rằng  và từ đó
 10
 5  4.10 16 ,  6  4.1032 .
Lời giải

Nhận xét 1 : xn  0, n  .

Nhận xét 2 : xn   , n 

.

Thật vậy: x1   .

1   1 
xn1   xn    .2. xn .   .
2 xn  2 xn

1   1   xn2 
a) Ta có: xn1  xn    x    0, n  (theo nhận xét 2 )
2  n xn  2 xn
16
 xn  giảm và bị chặn dưới.

  lim xn  L  L  0  .

1  
 L   L    L   L2    L  
2 L L

Vậy lim xn   .

1  
b)  n1  xn1     xn    
2 xn 

1  xn  2  xn   
2
 . 
2  xn 

 
2
xn    n2  n2
   (do xn   )
2 xn 2 xn 2 

 n2
  2    n 1 

2 22 2n
      
 n1   n    n1   ...   1 
      
2n
 
  n 1   .  1  .
 

c)   3, x1  2


1 2  3
 
2 3 2 3 

1

1  
 2 3 2 3 2 3 
4 3  6 10 
  5  4.10 2 ,  6  4.10 2 .
4 5

  xn   xn2
Câu 3.17. Lấy   1 , x1   và định nghĩa xn1   xn  .
1  xn 1  xn
a) Chứng minh x1  x3  x5  ... .
b) Chứng minh x2  x4  x6  ... .
c) Chứng minh rằng lim xn   .
d) So sánh tốc độ hội tụ của dãy này với dãy trong bài 3.16 .

17
Lời giải

Nhận xét 1 : xn  0, n  
 1 .
Thật vậy, ta có: x1    0 .

  xn
Giả sử  1 đúng tới n .    xn  0;1  xn  0  xn1   0.
1  xn

Nhận xét 2 : x2n   và x2 n1   n  


.  2
Thật vậy, ta có:

x2   
  x1
  
    x1 1    
1  x1 1  x1

 x2   
  1    x1  0x  .
1  x1 2

Giả sử  2  đúng tới n . Ta chứng minh x2 n 2   và x2 n1   .

  x2 n  1  1
Ta có x2 n1   1  1  
1  x2 n 1  x2 n 1 

 1  1
x2 n 2  1   1  .
1  x2 n  1 1 

  2  được chứng minh.

Ta có xn 2  1 
 1
 1
 1
 1
  1 xn  1 .
1  xn1  1 2 xn    1
1 1
xn  1

2 xn2  2
 xn 2  xn 
2 xn    1

x  x2 n  0 
  2 n 2 , n  .
 x 2 n  1  x2 n  1  0
Vậy a) và b) được chứng minh.

c) Ta có x2 k  tăng và bị chặn trên   lim x2 k  L1

  L1
 L1   L21    L1    mà x2 n  0, n  L1   .
1  L1
18
Tương tự, dãy x2 k 1  giảm và bị chặn dưới   lim x2 k 1  L2 .

  L2
 L2   L2   (do L2  0 ).
1  L2

 x2 k  và x2 k 1  cùng hội tụ về  suy ra lim xn   .


n

d) Đặt  n  xn  

  xn1    n 1  
  n    xn  
1  xn1 1   n 1  

   n 1    n1   n1  1  
 n       n 1  
1   n 1   1   n 1    1    
 n 1 

 1  1   1
  n   n 1   n 1 .   n 1 .
1     n 1 1  xn  1  x 
 2 

2n2
  1
  n  1 .
 1  x 
.
 2 

p 1 
Câu 3.18. Thay thế công thức đệ quy của bài 3.16 bằng xn1  xn  xn p 1 khi p là một số
p p
nguyên dương cố định, và mô tả kết quả của dãy xn  .

Lời giải

 p 1 
Nhận xét 1 : xn  0, n  (Do x1  0;  0,  0 )
p p

Nhận xét 2 : xn   , n 
p 
.

Thật vậy, ta có:

 p  1 xn   .xn p 1 
1
xn1 
p 


1

x  x  ...  xn   .xn p 1
p n n

1
 .p p xnp 1 . .xn p 1   .
p

Nhận xét 3 : xn  giảm.

19
Thật vậy, ta có:

xn1 p  1  .xn p p  1 
 
p
p
    . 1
xn p p p p

 xn1  xn , n  *
.

- Do xn  giảm và bị chặn dưới  xn  hội tụ.

  lim xn  L  L  0  .

p 1 
L .L  .L p 1
p p

1 
 .L  .L p 1  Lp    L   .
p

p p

p 1 
- xn1  .xn  .xn p 1
p p

 
xn     
  
 xn1    xn     p  1   xn   . 1 
p p p xn

 p x  p   x p
 1 
 
.
p  xn 
 


 
n n


p

Đặt y   1.
xn

 yp  1 
p

 xn1    xn   .  1 
p


 p  y
 1


 
 xn    1 

p

y p 1  y p 2  ...  y  1 
p

  p  1 y p 1  1 
  
 xn   .  1 
p

p


 


 xn   .
p
 p p 1 . 1  y 
p 1


 xn   .
p
 p p 1 .1  y  . 1  y  y 2
 ...  y p  2 

  . ppx 1  p  1
2
 xn  
p

20
 p  1
2

 
2
 xn  
p
.
p 
p

  x  p 
 n n

 p  1 .
2
Đặt 
  p
 p 

2 2n
    
  n1   .  n1   n   ...   1 
2
n
   
2n
 
  n1   .  1  , n  
.


n
Câu 3.19. Liên kết với từng chuỗi a   n  , trong đó  n là 0 hoặc 2 , số thực x  a    .
n 1 3n
Chứng minh rằng tập hợp tất cả x  a  chính là tập Cantor.
Lời giải

Gọi A là tập các x  a  định nghĩa như trên.

1 2
Trước hết ta thấy trong xây dựng tập Cantor trong bước đầu tiên, bỏ khoảng  ;  ra khỏi đoạn
3 3
1 2 1
 0;1 , ta sẽ chứng minh x   ;  có một khai triển tam phân thì phần tử đầu tiên sẽ là .
3 3 3

1
Giả sử x có một khai triển tam phần mà hạng tử đầu tiên không phải là .
3

0 2 2
 Phần tử đầu tiên chỉ có thể là bởi nếu là thì x  .
3 3 3

0 2 
Ta viết: x   2  ...  nn  ...
3 3 3

2 1 
2 
1  n 1 
2 2 3  3  1
Vậy x  2  ...  n  ...  lim  vô lí.
3 3 n 2 3
1
3

1
Vậy x phải có khai triển tam phân bắt đầu là .
3
Suy ra x  A .

21
1 0 2 2
Tuy nhiên có thể viết    ...  n  ...
3 3 9 3

2 2 0 0
   ...  n  ...
3 3 9 3

 1 2 
Như vậy, trong xây dựng tập Cantor P ở bước 1 ta có E  0;    ;1 là gồm các phần tử
3 3    
1
(nếu có) khai triển tam phân bắt đầu là .
3
Với cách lập luận như trên, tại bước thứ n khi xây dựng tập P ta có gồm tất cả các phần tử có
khai triển tam phân không có số 1 trên tử của n phần tử đầu tiên.

Cứ như vậy, ta được một tập Cantor gồm tất cả các phần tử có khai triển tam phân mà không có
tử số bằng 1 tại bất cứ vị trí nào, hay nói cách khác là tập A mà ta đang xét

Câu 3.20. Giả sử  pn  là một dãy Cauchy trong không gian metric X , và một số dãy con pni   hội tụ
đến một điểm p  X . Chứng minh rằng toàn bộ dãy  pn  hội tụ đến p .
Lời giải

Với   0 tùy ý.


Dãy pn   hội tụ đến p  M 
k
thỏa mãn: d pn , p   k
 2
, k  M .


Vì  pn  là dãy Cauchy  M  
thỏa mãn d pr , ps   2
, r , s  M  .

Chọn N  max  M , M   . Xét k  N .

 
   
 n  N : d  pn , p   d pn , pnk  d pnk , p 
2

2


Vậy dãy  pn  hội tụ đến p .

Câu 3.21. Chứng minh điều tương tự sau đây của định lí 3.10  b  : Nếu En  là một dãy các tập
đóng khác rỗng và bị chặn trong không gian metric X , nếu En  En1 và lim diamEn  0 thì
n

En bao gồm chính xác một điểm.
1

Lời giải

Chọn xn  En (cách chọn này hiển nhiên là có thể).

lim diamEn  0  xn  là dãy Cauchy.


n

22
Vì X đầy đủ nên x  X sao cho xn  x .

Khi đó x  En , n vì En đóng và chứa xn , m  n .


Tức x  En .
1

Giả sử y  x  En , n .

Khi đó d  x , y   0 .

Vì lim diamEn  0 có nghĩa là với n đủ lớn thì diamEn  d  x , y  tức x  En hoặc y  En (vô lí).
n


Vậy x là điểm duy nhất thuộc En .
1

Câu 3.22. Giả sử X là một không gian metric đầy đủ, khác rỗng, và Gn  là một dãy con mở trù mật

của X . Chứng minh định lí Baire, tức là, Gn khác rỗng (rõ hơn, nó còn trù mật trong X ).
n 1

Lời giải

Lấy Fn là phần bù của Gn .

 Fn là đóng và không bao gồm tập mở nào trong đó.

Ta chứng minh bất kì những tập U nào mở, khác rỗng sẽ chứa một điểm không thuộc Fn , nên
điểm đó sẽ thuộc Gn .

Do U mở  U  F1  x1 : x1  U \ F1 .

Do U \ F1 mở (nếu U \ F1 đóng thì F1  U \ F1   U đóng, vô l )

 r1  0 : B1  x1 , r1   U \ F1

 r1 
Lấy E1  x1 ,   E1  B1  U \ F1
 2

Do E1 là hình cầu mở  E1  F2  x2 : x2  E1 \ F2 .

Do E1 \ F2 mở  r2  0 : B2  x2 , r2   E1 \ F2

 B2  U \ F1  F2 

 r2 
Lấy E2  x2 ,   E2  B2  U \ F1  F2 
 2

23
Nên E2  E1 .

Tiếp tục quá trình, ta xây dựng dãy các hình cầu mở E j mà Ej  Ej  Ej 1 và

diamEj  diamEj  0 .

Theo 3.21  x : x  Ej

 x  U \ F1  F2  ...  Fn 

 
 x  U   Gn 
 n 1 

 Gn khác rỗng.
n 1

Câu 3.23. Giả sử  pn  và qn  là các chuỗi Cauchy trong không gian metric X . Chứng minh rằng dãy

d  p , q  hội tụ.
n n

Lời giải

m, n  :

d  pn , q n   d  p n , p m   d  p m , q m   d  q m , q n 

 d  pn , q n   d  p m , q m   d  p n , p m   d  q m , q n   1 

d  p m , q m   d  p m , pn   d  pn , q n   d  q n , q m 

 d  p m , q m   d  pn , q n   d  p m , p n   d  q m , q n   2 

Từ  1 và  2  :  d  pn , qn   d  pm , qm   d  pn , pm   d qm , qn  ,  m, n  .  3
p  và q  là các chuỗi Cauchy nên với   0 tùy ý ta có:
n n


N1 đủ lớn thỏa mãn: d  pn , pm   , n, m  N1 .
2


N 2 đủ lớn thỏa mãn d  qn , qm   , n, m  N 2 .
2

Chọn N  max  N1 , N 2 

 d  pn , pm   d  qn , qm    , m , n  N .

Từ  3   d  pn , qn   d  pm , qm    , n, m  N

24
 
 d  pn , qn  hội tụ.

Câu 3.24. Cho X là một không gian metric.


a) Gọi hai chuỗi Cauchy  pn  và qn  trong X là tương đương nếu: lim d pn , qn  0 . Chứng
n
 
minh rằng đó là một quan hệ tương đương.
b) Lấy X là tập của tất cả các lớp tương đương thu được. Nếu P  X , Q  X  , p   P , n

q   Q , định nghĩa   P , Q   lim d  p , q


n
n n n  . Chứng minh rằng nếu ta thay p  và q  bằng
n n

những dãy tương đương thì   P , Q  không đổi.

c) Chứng minh kết quả không gian metric X đầy đủ.


d) Với mỗi p  X , có một dãy Cauchy các điều kiện là p ; lấy Pp là một bộ phận của X bao

   
hàm dãy này. Chứng minh rằng  Pp , Pq  d p , q với mọi p , q  X . Nói cách khác, ánh xạ 

định nghĩa bởi   p   Pp là một phép đẳng cự của X vào trong X .

e) Chứng minh rằng   X  là trù mật trong X , và   X   X  nếu X là đầy đủ. Theo d), ta có

thể định danh X và   X  và ta coi X bị lồng vào không gian metric đầy đủ X . Ta gọi X là
bổ sung của X .
Lời giải

a) Ta cần chứng minh đây là quan hệ tương đương.

+) Tính phản xạ: hiển nhiên  pn  phải tương đương với chính nó vì lim d pn , pn  0 .
n
 
+) Tính đối xứng: nếu  pn  tương đương với qn  thì qn  tương đương với  pn  vì

lim d  pn , qn   0  lim d  qn , pn   0 .
n n

+) Tính bắc cầu: giả sử  pn  tương đương với qn  và qn  tương đương với un  .

 lim d  pn , qn   0 và lim d  qn , un   0
n n


Xét lim d pn , un :
n

 
Vì d  pn , un   d  pn , qn   d  qn , un  mà d  pn , un   0 và lim d  pn , qn   d  qn , un   0
n

 lim d  pn , un   0 .
n

Vậy  pn  tương đương với un  .

b) Ta cần chứng minh nếu ta thay  pn  và qn  bằng những dãy tương đương thì   P , Q 
không đổi.
25
   
Lấy pn  P , qn  Q .

Giả sử lim d  pn , qn   d .

  0, N , n  N :

    
d pn , qn  d  d pn , pn  d  pn , qn   d qn , qn  d   . 
  
Vậy lim d pn , qn  lim d pn , qn  d   P ,Q .
n n
  
Ta cần chứng minh  là một khoảng cách trong X .

+) Hiển nhiên   0 .

Giả sử   P , Q   0 , lấy pn  P , qn  Q  lim d p n , q n  0 .


     
n

 pn tương đương với q n , tức pn  P tương đương qn  Q hay P  Q .

+) Hiển nhiên   P , Q     Q , P  , thật vậy:

Với  pn   P , qn   Q thì lim d pn , qn  lim d qn , pn .


   
n n

+) Cho P , Q ,U  X  , Q  X  .

Lấy  pn   P , qn   Q , un   U .

Ta có: d  pn , un   d  pn , qn   d  qn , un 

 lim d  pn , un   lim d  pn , qn   lim d  qn , un 


n n n

   P ,U     P , Q    Q ,U  .

Vậy  là một khoảng cách trong X .

c) Để chứng minh X đầy đủ, ta cần chứng minh: mọi dãy Cauchy trong X đều hội tụ trong X .

Cho  pk  là dãy Cauchy trong X , chọn  pkn  là dãy Cauchy trong X sao cho

p   P , k  1, 2,... .
kn k

Với mỗi k , cho N k là số nguyên dương sao cho d  pkn , dkm   2  k .

 
Đặt pk  pkNk  d pk , pkn  2  k , n  N k  lim d pk , pkn  2  k
n
 
k , l , n ta có: d  pk , pl   d  pk , pkn   d  pkn , pln   d  pln , pl  .

26
Lấy n đủ lớn, giả sử k  l , ta được:

d  pk , pl   2  k    pk , pl   2  l  2.2  k    pk , pl 

  pk  là dãy Cauchy.

Đặt P  X sao cho  pk   P , ta có:

n n
 
  pk , p   lim d  pkn , pn   lim d  pkn , pk   d  pk , pn   2  k  2.2  k  lim sup   pk , pn  .
n

 
Với   0 , chọn N1  log 2 3  log 2  , N 2 sao cho   pk , pl  , k  N 2 , l  N 2 .
 2 

Chọn N  max  N1 , N 2  .

Với k  N , khi đó   pk , p    nghĩa là  pk  hội tụ đến p trong X .

Tức X đẩy đủ.

d) Vì lim pn  p ,lim qn  q .

p   p : q   P , ta có:   P , P   lim d  p
n p n q p q n n
, qn   d  p , q  .

Khi đó  : X  X  là một ánh xạ đẳng cự.

p pq .

e) +) Chứng minh   x  trù mật trong X

 với P bất kì thuộc X , p  X thỏa mãn  p , Pp   .  


Lấy  pn   P và  pn  là dãy Cauchy.


 N , n, m  N ta có: d  pn , pm   .
2


 
Chọn p  pN 1 , khi đó  P , Pp  lim pn , p 
n
  2
.

Vậy   x  trù mật trong X .

+) Chứng minh: nếu X đầy đủ thì   X   X  .

Vì X đầy đủ, khi đó:

P  X  , pn   P , p  X : Pn  p .

27
Mọi dãy tương đương với  pn  thì đều hội tụ đến p do đó P  Pp .

Do đó   X   X  khi X đầy đủ.

+) Cần phải chứng minh rằng X đó là duy nhất, có nghĩa là: Nếu Y và Z là hai không gian
metric đầy đủ bất kì, mỗi không gian chứa một tập trù mật, đẳng cự với X thì Y đẳng cự với Z

Cho  và  lần lượt là đằng cự từ X  Y và Y  Z .

  X  trù mật trong Y ,   X  trù mật trong Z .

y  Y , xn   X sao cho   xn   y .

Mặt khác, xn  là dãy Cauchy trong X    xn  là dãy Cauchy trong Z . Vì Z đầy đủ nên

z  Z ,  xn   z .

Ta định nghĩa tương ứng   y   z suy ra tương ứng trên không phụ thuộc vào đại diện.

   
Giả sử  xn  Y , xn  y , khi đó:

    d  x , x    d   x  ,  x   0    x    z .
dZ   xn  , xn X n n Y n n n

+)  là một đẳng cự?

 
Giả sử y1  lim  x1n , y2  lim  x2 n .
n n
 
     
 dZ   y1  ,  y2   lim dz   x1n  ,  x2 n   lim dX  x1n , x2 n   lim dY   x1n  ,   x2 n   dY  y1 , y2 

Vậy   Y   Z , bởi lẽ tương tự ta cũng định nghĩa một ánh xạ ngược  : Z  Y bằng những bước
như trên.

Câu 3.25. Lấy X là không gian metric mà mọi điểm đều là số hữu tỉ, với khoảng cách d  x , y   x  y .
Bổ sung của không gian này là gì?
Lời giải

Theo bài 3.24 , tập đầy đủ của không gian X là bất kì không gian metric đầy đủ nào chứa một
tập con trù mật đẳng cự với không gian X .

Vì trù mật trong , một dãy Cauchy các số hữu tỉ đều hội tụ tới một số thực duy nhất.

Và hiển nhiên, hai dãy tương đương khi c ng hội tụ đến cùng một số thực.

Do đó, chính là tập đầy đủ của .

28

You might also like